Đến nội dung

Hình ảnh

Topic về bất đẳng thức

* * * * * 16 Bình chọn

  • Please log in to reply
Chủ đề này có 206 trả lời

#61
alex_hoang

alex_hoang

    Thượng úy

  • Hiệp sỹ
  • 1152 Bài viết

Thấy BĐT này nhìn cũng "bắt mắt", đưa lên mọi người xem (dạo này bên box BĐT vắng quá)

Bài 31$a,b,c>0$. Chứng minh rằng

$\sqrt[3]{\dfrac{a^2+bc}{b^2+c^2}}+\sqrt[3]{\dfrac{b^2+ac}{a^2+c^2}}+\sqrt[3]{\dfrac{c^2+ba}{b^2+a^2}} \ge \dfrac{9\sqrt[3]{abc}}{a+b+c}$


P/s: Ai đã biết lời giải có sẵn xin đừng post lên, hãy để người khác suy nghĩ.

Đểcho bất đẳng thức không bị phân tán đi quá nhiều topic mình post lời giải bài toán của Toàn tại đây và coi như bài 31
Giải
Bất đẳng thức tương đương
$\sum {\dfrac{{{a^2} + bc}}{{\sqrt[3]{{abc({a^2} + bc)({b^2} + {c^2})}}}}} \ge \dfrac{9}{{a + b + c}}$
Sử dụng AM GM ta có
$\sum {\dfrac{{{a^2} + bc}}{{\sqrt[3]{{abc({a^2} + bc)({b^2} + {c^2})}}}}} \ge \dfrac{{3({a^2} + bc)}}{{\sum\limits_{sym} {{a^2}b} }}$
Vậy ta phải chứng minh
$\dfrac{{3({a^2} + bc)}}{{\sum\limits_{sym} {{a^2}b} }} \ge \dfrac{9}{{a + b + c}} \Leftrightarrow {a^3} + {b^3} + {c^3} + 3abc \ge \sum\limits_{sym} {{a^2}b} $
alex_hoang


HẸN NGÀY TRỞ LẠI VMF THÂN MẾN

http://www.scribd.co...oi-Ban-Cung-The

#62
vietfrog

vietfrog

    Trung úy

  • Hiệp sỹ
  • 947 Bài viết
Topic sôi đông thật. Mỗi Box đều có vài Topic về BĐT. Mọi người cần post tập trung để tránh phân tán.
Góp vui một bài:
Bài 32:
Giả sử hệ phương trình: $\left\{ \begin{array}{l}{x^2} + xy + {y^2} = 3\\{y^2} + yz + {z^2} = 16\end{array} \right.$ có nghiệm.

Chứng minh rằng:

$xy + yz + zx \le 8$



Bài viết đã được chỉnh sửa nội dung bởi vietfrog: 08-09-2011 - 22:55

Sống trên đời

Cần có một tấm lòng

Để làm gì em biết không?

Để gió cuốn đi...

Chủ đề:BĐT phụ

HOT: CÁCH VẼ HÌNH


#63
HÀ QUỐC ĐẠT

HÀ QUỐC ĐẠT

    Thượng sĩ

  • Thành viên
  • 295 Bài viết

Topic sôi đông thật. Mỗi Box đều có vài Topic về BĐT. Mọi người cần post tập trung để tránh phân tán.
Góp vui một bài:
Bài 32:
Giả sử hệ phương trình: $\left\{ \begin{array}{l}{x^2} + xy + {y^2} = 3\\{y^2} + yz + {z^2} = 16\end{array} \right.$ có nghiệm.

Chứng minh rằng:

$xy + yz + zx \le 8$



Ta có:
$ 48=( x^{2}+xy+ y^{2}).( y^{2}+yz+ z^{2})=[ (y+\dfrac{x}{2}) ^{2}+(\dfrac{x\sqrt{3} }{2}) ^{2}].[(\dfrac{z\sqrt{3} }{2}) ^{2}+ (y+\dfrac{z}{2}) ^{2}]$
$\geq [ (\dfrac{yz\sqrt{3} }{2}+\dfrac{xz\sqrt{3} }{4}+\dfrac{xy\sqrt{3} }{2}+\dfrac{xz\sqrt{3} }{4}) ^{2}]$
$ = (\dfrac{(xy+yz+zx)\sqrt{3} }{2}) ^{2} \Leftrightarrow xy+yz+zx \leq 8$

Bài viết đã được chỉnh sửa nội dung bởi dark templar: 11-09-2011 - 21:51
Gõ cách dòng cho dễ nhìn.


#64
alex_hoang

alex_hoang

    Thượng úy

  • Hiệp sỹ
  • 1152 Bài viết
Mong các bạn trao đổi tích cực hơn trong topic này
Bài 33Cho các số thực không âm a,b,c sao cho a+b+c=2.CMR
$\left( {{a^2} + {b^2}} \right)\left( {{b^2} + {c^2}} \right)\left( {{c^2} + {a^2}} \right) \le 2$
Nguồn:Mathlink.ro

Bài viết đã được chỉnh sửa nội dung bởi alex_hoang: 10-09-2011 - 17:11

alex_hoang


HẸN NGÀY TRỞ LẠI VMF THÂN MẾN

http://www.scribd.co...oi-Ban-Cung-The

#65
dark templar

dark templar

    Kael-Invoker

  • Hiệp sỹ
  • 3788 Bài viết
Bài 34: Cho $n \in N^*;a=const>1$.Chứng minh rằng:

$\sum\limits_{k=1}^{n}\dfrac{k}{a^{k}}<\dfrac{a}{(a-1)^2}$


Bài 35: Chứng minh rằng:$a(a+b)^3+b(b+c)^3+c(c+a)^3 \ge 0,\forall a,b,c \in R$
"Do you still... believe in me ?" Sarah Kerrigan asked Jim Raynor - Starcraft II:Heart Of The Swarm.

#66
HÀ QUỐC ĐẠT

HÀ QUỐC ĐẠT

    Thượng sĩ

  • Thành viên
  • 295 Bài viết

Bài 34: Cho $n \in N^*;a=const>1$.Chứng minh rằng:

$\sum\limits_{k=1}^{n}\dfrac{k}{a^{k}}<\dfrac{a}{(a-1)^2}$


Bài 35: Chứng minh rằng:$a(a+b)^3+b(b+c)^3+c(c+a)^3 \ge 0,\forall a,b,c \in R$

Em thử giải thế này,nếu sai xin mọi người góp ý:
$\begin{array}{l}
{\rm{x\'e t a,b,c}} \in {\rm{[0;1] ta c\'o }}:ab(ab - 1) \le 0 \Leftrightarrow {a^2}{b^2} \le ab \\
\Rightarrow {a^2}{b^2} + {b^2}{c^2} + {c^2}{a^2} \le ab + bc + ca \\
C{\rm{\'o }}:VT = ({a^2} + {b^2} + {c^2})({a^2}{b^2} + {b^2}{c^2} + {c^2}{a^2}) - {a^2}{b^2}{c^2} \\
\Leftrightarrow VT \le ({a^2} + {b^2} + {c^2})(ab + bc + ca) = ({a^2} + {b^2} + {c^2}){\rm{[}}2 - \dfrac{{({a^2} + {b^2} + {c^2})}}{2}{\rm{]}} \le 2 \\
\Leftrightarrow ({a^2} + {b^2} + {c^2}){\rm{[}}4 - ({a^2} + {b^2} + {c^2}){\rm{]}} \le {\rm{4}} \\
\Leftrightarrow {({a^2} + {b^2} + {c^2} - 2)^2} \ge 0 \\
\\
X{\rm{\'e }}ta,b,c \in {\rm{[}}1;2]c{\rm{\'o }}:{a^2} + {b^2} + {c^2} \ge a + b + c \\
(ab - 1)(ab - 2) \le 0 \Leftrightarrow {a^2}{b^2} \le 3ab - 2 \\
\Rightarrow {a^2}{b^2} + {b^2}{c^2} + {c^2}{a^2} \le 3(ab + bc + ca) - 6 \\
C{\rm{\'o }}:VT = ({a^2} + {b^2} + {c^2})({a^2}{b^2} + {b^2}{c^2} + {c^2}{a^2}) - {a^2}{b^2}{c^2} \\
\Leftrightarrow VT \le ({a^2} + {b^2} + {c^2}){\rm{[}}3(ab + bc + ca) - 6{\rm{]}} \\
\Leftrightarrow VT \le ({a^2} + {b^2} + {c^2}){\rm{[}}6 - \dfrac{{3({a^2} + {b^2} + {c^2})}}{2}{\rm{ - 6]}} \le 2 \\
\Leftrightarrow - \dfrac{{3{{({a^2} + {b^2} + {c^2})}^2}}}{2} \le 2 \\
\end{array}$

#67
HÀ QUỐC ĐẠT

HÀ QUỐC ĐẠT

    Thượng sĩ

  • Thành viên
  • 295 Bài viết
Bài 33
Em thử giải thế này,nếu sai xin mọi người góp ý:

$\begin{array}{l}
{\rm{x\'e t a,b,c}} \in {\rm{[0;1] ta c\'o }}:ab(ab - 1) \le 0 \Leftrightarrow {a^2}{b^2} \le ab \\
\Rightarrow {a^2}{b^2} + {b^2}{c^2} + {c^2}{a^2} \le ab + bc + ca \\
C{\rm{\'o }}:VT = ({a^2} + {b^2} + {c^2})({a^2}{b^2} + {b^2}{c^2} + {c^2}{a^2}) - {a^2}{b^2}{c^2} \\
\Leftrightarrow VT \le ({a^2} + {b^2} + {c^2})(ab + bc + ca) = ({a^2} + {b^2} + {c^2}){\rm{[}}2 - \dfrac{{({a^2} + {b^2} + {c^2})}}{2}{\rm{]}} \le 2 \\
\Leftrightarrow ({a^2} + {b^2} + {c^2}){\rm{[}}4 - ({a^2} + {b^2} + {c^2}){\rm{]}} \le {\rm{4}} \\
\Leftrightarrow {({a^2} + {b^2} + {c^2} - 2)^2} \ge 0 \\
\\
X{\rm{\'e }}ta,b,c \in {\rm{[}}1;2]c{\rm{\'o }}:{a^2} + {b^2} + {c^2} \ge a + b + c \\
(ab - 1)(ab - 2) \le 0 \Leftrightarrow {a^2}{b^2} \le 3ab - 2 \\
\Rightarrow {a^2}{b^2} + {b^2}{c^2} + {c^2}{a^2} \le 3(ab + bc + ca) - 6 \\
C{\rm{\'o }}:VT = ({a^2} + {b^2} + {c^2})({a^2}{b^2} + {b^2}{c^2} + {c^2}{a^2}) - {a^2}{b^2}{c^2} \\
\Leftrightarrow VT \le ({a^2} + {b^2} + {c^2}){\rm{[}}3(ab + bc + ca) - 6{\rm{]}} \\
\Leftrightarrow VT \le ({a^2} + {b^2} + {c^2}){\rm{[}}6 - \dfrac{{3({a^2} + {b^2} + {c^2})}}{2}{\rm{ - 6]}} \le 2 \\
\Leftrightarrow - \dfrac{{3{{({a^2} + {b^2} + {c^2})}^2}}}{2} \le 2 \\
\end{array}$

#68
HÀ QUỐC ĐẠT

HÀ QUỐC ĐẠT

    Thượng sĩ

  • Thành viên
  • 295 Bài viết
Bài 33
Em thử giải thế này,nếu sai xin mọi người góp ý:
*Xét a,b,c :in [0;1] ta có:ab(ab-1) :-? 0 -_- $ a^{2}b^{2}\leq ab$
:Rightarrow $ a^{2}b^{2}+b^{2}c^{2}+c^{2}a^{2}\leq ab+bc+ca$
Ta có:$ VT=( a^{2}+ b^{2}+ c^{2})(a^{2}b^{2}+b^{2}c^{2}+c^{2}a^{2})- a^{2} b^{2}c^{2}\leq ( a^{2}+ b^{2}+ c^{2})(ab+bc+ca)= ( a^{2}+ b^{2}+ c^{2})(2-\dfrac{( a^{2}+ b^{2}+ c^{2})}{2}\leq 2$
-_- $( a^{2}+ b^{2}+ c^{2})[4-( a^{2}+ b^{2}+ c^{2})]\leq 4$
:Leftrightarrow $ (a^{2}+ b^{2}+ c^{2}-2)^{2}\geq 0 (Đúng) $
Dấu"=" xảy ra :Leftrightarrow a=b=1;c=0 và các hoán vị
*Xét a,b,c :in [1;2] có $a^{2}+ b^{2}+ c^{2}\geq a+b+c=2$
(ab-1)(ab-2) ;) 0 :Leftrightarrow$ a^{2}b^{2}\leq 3ab-2\Leftrightarrow a^{2}b^{2}+b^{2}c^{2}+c^{2}a^{2}\leq 3(ab+bc+ca)-6$
$ VT=( a^{2}+ b^{2}+ c^{2})(a^{2}b^{2}+b^{2}c^{2}+c^{2}a^{2})- a^{2}b^{2}c^{2}$
$\Leftrightarrow VT\leq (a^{2}+b^{2}+c^{2})[3(ab+bc+ca)-6]=\dfrac{-3( a^{2}+ b^{2}+ c^{2}) ^{2} }{2}\leq 2 (Đúng) $
Vậy bất đẳng thức được chứng minh. Dấu"=" xảy ra :Leftrightarrow a=b=1;c=0 và các hoán vị

Bài viết đã được chỉnh sửa nội dung bởi HÀ QUỐC ĐẠT: 12-09-2011 - 17:47


#69
Tạ Hồng Quảng

Tạ Hồng Quảng

    Binh nhì

  • Thành viên
  • 16 Bài viết
Bài 36 (Sáng tác):
Cho $ a, \, b \, ,\, c \, \ge \, 0 $ thoả mãn $ a+b+c+abc \le 4 $ . Chứng minh rằng :

$ \sqrt[3]{a}+\sqrt[3]{b}+\sqrt[3]{c}\ge \sqrt[3]{ab}+\sqrt[3]{bc}+\sqrt[3]{ca} $

#70
alex_hoang

alex_hoang

    Thượng úy

  • Hiệp sỹ
  • 1152 Bài viết
Bài 37.Cho $ a,b,c>0$ và $ abc=1 $. Chứng minh rằng:
$ \dfrac{1}{\sqrt{4a^2+a+4}}+\dfrac{1}{\sqrt{4b^2+b+4}}+\dfrac{1}{\sqrt{4c^2+c+4}} \le 1 $

Bài viết đã được chỉnh sửa nội dung bởi alex_hoang: 20-09-2011 - 09:38

alex_hoang


HẸN NGÀY TRỞ LẠI VMF THÂN MẾN

http://www.scribd.co...oi-Ban-Cung-The

#71
vietfrog

vietfrog

    Trung úy

  • Hiệp sỹ
  • 947 Bài viết
Góp một bài.
Bài 38:Cho $a,b,c,x,y,z$ là các số thực dương. Biết : $a+b+c=1$
Chứng minh rằng:
\[ax + by + cz + 2\sqrt {(xy + yz + xz)(ab + ac + bc)} \le x + y + z\]

Bài viết đã được chỉnh sửa nội dung bởi bboy114crew: 30-09-2011 - 11:20

Sống trên đời

Cần có một tấm lòng

Để làm gì em biết không?

Để gió cuốn đi...

Chủ đề:BĐT phụ

HOT: CÁCH VẼ HÌNH


#72
Crystal

Crystal

    ANGRY BIRDS

  • Hiệp sỹ
  • 5534 Bài viết
Một cách khác cho bài 32. Xem chi tiết tại đây:

http://diendantoanhoc.net/index.php?showtopic=44666&st=60

#73
HÀ QUỐC ĐẠT

HÀ QUỐC ĐẠT

    Thượng sĩ

  • Thành viên
  • 295 Bài viết

Bài 36 (Sáng tác):
Cho $ a, \, b \, ,\, c \, \ge \, 0 $ thoả mãn $ a+b+c+abc \le 4 $ . Chứng minh rằng :

$ \sqrt[3]{a}+\sqrt[3]{b}+\sqrt[3]{c}\ge \sqrt[3]{ab}+\sqrt[3]{bc}+\sqrt[3]{ca} $


$\sqrt[3]{a}=x;\sqrt[3]{b}=y;\sqrt[3]{c}=z$
Bất đẳng thức cần chứng minh:$\Leftrightarrow x+y+z\geq xy+yz+zx$
Từ giả thiết ta có:
$x^{3}+y^{3}+z^{3}+x^{3}y^{3}z^{3}\leq 4\Leftrightarrow xyz\leq 1$
$*x+y+z\leq 3$
$\Leftrightarrow 3p^{2}\geq p^{3}\geq 3pq$
$\Leftrightarrow p\geq q$
$*x+y+z\geq 3$
Áp dụng bất đẳng thức schur ta có:
$9r\geq 4pq-p^{3}\Leftrightarrow p^{3}+9r-3pq\geq pq$
có:$4\geq x^{3}+y^{3}+z^{3}+x^{3}y^{3}z^{3}$
$=p^{3}-3pq+9r-6r+r^{3}\geq pq+r^{3}-6r\geq pq-5(vì r \leq 1)$
$\Leftrightarrow 9\geq pq$
$Mà p^{2}\geq 9\Leftrightarrow p^{2}\geq pq\Leftrightarrow p\geq q$
Dấu "="xảy ra khi x=y=z=1 hay a=b=c=1

Bài viết đã được chỉnh sửa nội dung bởi HÀ QUỐC ĐẠT: 06-10-2011 - 22:32


#74
NguyThang khtn

NguyThang khtn

    Thượng úy

  • Hiệp sỹ
  • 1468 Bài viết
\[ax + by + cz + 2\sqrt {(xy + yz + xz)(ab + ac + bc)}\]
$ \leq \sqrt{(a^2+b^2+c^2)(x^2+y^2+z^2)} +\sqrt {2(xy + yz + xz)2(ab + ac + bc)} $
$ \leq \sqrt{(a+b+c)^2.(x+y+z)^2} = x+y+z$(do $1=a+b+c$)

Bài viết đã được chỉnh sửa nội dung bởi bboy114crew: 12-10-2011 - 12:20

It is difficult to say what is impossible, for the dream of yesterday is the hope of today and the reality of tomorrow

 


#75
Ngày không em

Ngày không em

    Binh nhì

  • Thành viên
  • 10 Bài viết
Bài 39 Cho các số thực dương $ a,b,c $thay đổi thỏa mãn $ abc=4 $.Tìm giá trị nhỏ nhất của biểu thức
$$S = \dfrac{{{a^3}}}{{\sqrt {(1 + {a^4}\sqrt a )(1 + {b^4}\sqrt b )} }} + \dfrac{{{b^3}}}{{\sqrt {(1 + {c^4}\sqrt c )(1 + {b^4}\sqrt b )} }} + \dfrac{{{c^3}}}{{\sqrt {(1 + {a^4}\sqrt a )(1 + {c^4}\sqrt c )} }}$$

Bài viết đã được chỉnh sửa nội dung bởi alex_hoang: 27-10-2011 - 00:09


#76
alex_hoang

alex_hoang

    Thượng úy

  • Hiệp sỹ
  • 1152 Bài viết
Bài 40Chứng minh rằng với mọi số thực dương $a, b, c$, ta luôn có
$$\dfrac{a^2+b^2+c^2}{ab+bc+ca} \ge \dfrac{ab}{a^2+bc+ca}+\dfrac{bc}{b^2+ca+ab}+\dfrac{ca}{c^2+ab+bc}$$
alex_hoang


HẸN NGÀY TRỞ LẠI VMF THÂN MẾN

http://www.scribd.co...oi-Ban-Cung-The

#77
taminhhoang10a1

taminhhoang10a1

    Trung sĩ

  • Thành viên
  • 131 Bài viết
Bài 41, Cho a,b,c không âm. CMR:

$\dfrac{{a(b + c)}}{{b^2 + bc + c^2 }} + \dfrac{{b(c + a)}}{{c^2 + ca + a^2 }} + \dfrac{{c(a + b)}}{{a^2 + ab + b^2 }} \ge 2$
THPT THÁI NINH - THÁI THỤY - THÁI BÌNH

#78
NguyThang khtn

NguyThang khtn

    Thượng úy

  • Hiệp sỹ
  • 1468 Bài viết


Bài 41, Cho a,b,c không âm. CMR:

$\dfrac{{a(b + c)}}{{b^2 + bc + c^2 }} + \dfrac{{b(c + a)}}{{c^2 + ca + a^2 }} + \dfrac{{c(a + b)}}{{a^2 + ab + b^2 }} \ge 2$

Ap dụng BDT Cauchy-chwarz:
$[\sum \dfrac{{a(b + c)}}{{b^2 + bc + c^2 }}][\sum \dfrac{{a(b^2 + bc + c^2)}}{{b + c}}] \geq (a+b+c)^2$
Ta sẽ CM:
$(a+b+c)^2 \geq \sum \dfrac{{a(b^2 + bc + c^2)}}{{b + c}}$
Do $\dfrac{{a(b^2 + bc + c^2)}}{{b + c}} = ab+ac - \dfrac{abc}{b+c}$
BDT trên tương đương với:
$(a+b+c)^2 +2abc\sum \dfrac{1}{b+c} \geq 4(ab+bc+ac)$
$\Leftrightarrow 2(a^2+b^2+c^2) + 2abc\sum \dfrac{1}{b+c} \geq (a+b+c)^2$
Ta lại có:
$2a^2+\dfrac{2abc}{b+c} =\dfrac{{2a(ab + bc + câ)}}{{b + c}}$
Nên BDT tương đương với:
$\sum \dfrac{a}{b+c} \geq \dfrac{(a+b+c)^2}{2(ab+bc+ac)}$
Đúng theo Cauchy-chwarz.
p\s: Bài 40 Em làm đến đoạn :
CM:
$\sum\dfrac{a(a^2-b^2)(a+c)}{ab+bc+ac} \geq 0 $
Mà chả biết C-S kiểu gì cả!

Bài viết đã được chỉnh sửa nội dung bởi bboy114crew: 27-10-2011 - 15:15

It is difficult to say what is impossible, for the dream of yesterday is the hope of today and the reality of tomorrow

 


#79
alex_hoang

alex_hoang

    Thượng úy

  • Hiệp sỹ
  • 1152 Bài viết
Bài 40 đó em thử sử dụng Cauchy Schwaz
VD:$({a^2} + bc + ca)({b^2} + bc + ca) \ge {(ab + bc + ca)^2}$
alex_hoang


HẸN NGÀY TRỞ LẠI VMF THÂN MẾN

http://www.scribd.co...oi-Ban-Cung-The

#80
alex_hoang

alex_hoang

    Thượng úy

  • Hiệp sỹ
  • 1152 Bài viết
Bài 42Cho $a,b,c$ là các số thực dương thỏa mãn $abc=1$. Chứng minh rằng:
$\dfrac{(3a-1)^2}{2a^2+1}+\dfrac{(3b-1)^2}{2b^2+1}+\dfrac{(3c-1)^2}{2c^2+1}\geq 4$
alex_hoang


HẸN NGÀY TRỞ LẠI VMF THÂN MẾN

http://www.scribd.co...oi-Ban-Cung-The




0 người đang xem chủ đề

0 thành viên, 0 khách, 0 thành viên ẩn danh